LSAT and Law School Admissions Forum

Get expert LSAT preparation and law school admissions advice from PowerScore Test Preparation.

 Administrator
PowerScore Staff
  • PowerScore Staff
  • Posts: 8919
  • Joined: Feb 02, 2011
|
#23947
Complete Question Explanation

Assumption. The correct answer choice is (B)

This passage is basically a critique of political campaign coverage, based on the claim that the press reports on advisors, and the process of the campaign, rather than on the candidates and their positions on substantive issues. The author concludes that voters lose out by being deprived of information necessary to informed decision making.

Answer choice (A): While the author does draw the analogy to chess, it is not assumed that this is the most appropriate analogy. If we negate this answer choice, it has no effect on the strength of the argument.

Answer choice (B): This is the correct answer choice. It is the supporter assumption that the candidates are indeed taking substantive policy positions. Without this assumption, the argument fails: If the candidates aren’t taking substantive positions, the voters aren’t being deprived of information necessary for informed decision making.

Answer choice (C): The argument does not assume that substantive issues are determined by the press, but rather that there are substantive candidate positions which the press chooses not to cover.

Answer choice (D): The inability to make informed decision is argued to be based on poor press coverage, not assumed to be based on lack of voter attention to the election.

Answer choice (E): This is an opposite answer—the author specifically distinguishes these two types of reporting, arguing that one is preferable to the other in political campaign reporting.
 reg4315
  • Posts: 12
  • Joined: Sep 13, 2017
|
#40426
I was between B and D, and ultimately chose D simply because I felt like B was already given in the stimulus. By that I mean I took the final line of the stimulus to be true. That being -

"it is clear that the campaign advisors should...and let the press report on the most revealing positions on substantive issues the candidates have taken."

The stimulus is already telling us that candidates "have [already] taken" positions on substantive issues - aren't we supposed to take what's given in the stimulus as fact? If this final line didn't exist, I would have found B to be the perfect answer.

I guess the more important question here is, how can I determine that that point made is not necessarily given, but rather that there is an underlying assumption that the statement made by the author is true? i.e., rather than just taking that as true in the first place.

I feel like this comes up a lot so any help on this would be greatly appreciated! Thank you!
 Eric Ockert
PowerScore Staff
  • PowerScore Staff
  • Posts: 164
  • Joined: Sep 28, 2011
|
#40432
Even though answer choice (B) appears as though it were already stated, look closely at the language. It reads:
"... let the press report on the most revealing positions on substantive issues the candidates have taken."
This statement only refers to the most revealing statements they have made, but doesn't actually claim they've made any. If I were, for example, to claim that the press should report on the most accurate statements made by a particular candidate, yet the candidate only tells lies, then I can't argue the press should really be focusing on any statements.

The same is true here. If the candidates haven't actually taken any positions on the issues, you can't really focus on the most revealing of those positions. So, for this argument to make sense, the author had to assume that there were at least some positions that were taken.

Hope that helps!
 reg4315
  • Posts: 12
  • Joined: Sep 13, 2017
|
#40838
Ah, I see. That makes sense! Thank you!
 marieallen
  • Posts: 16
  • Joined: Jan 31, 2017
|
#44419
OK - I'm just totally confused here. In QQT I for LR, Pg 23 #10, the answer key says the correct answer is E. However, when I input the first 6 words of the stimulus into "search," I found two different answers to this stimulus. I do note that one "Administrator" response refers to the stimulus as Question 14, while the other response refers to Question 13. The two responses provide two different answers to the stimulus: one agreeing with the answer key as E (reference Question 14) and the other (reference Question 13) stating the correct answer is B (which I really disagree with...!). Can someone clarify? Thanks.
User avatar
 Stephanie Oswalt
PowerScore Staff
  • PowerScore Staff
  • Posts: 805
  • Joined: Jan 11, 2016
|
#44433
Hi Marie!

There's actually 2 questions with the same stimulus on the June 1991 test. :-D

The answer to #13 "The press reports on political campaigns these days as..." (which this thread is referring to) is B.
The answer to #14 "The press reports on political campaigns these days as..." (which you're referring to) is E.

I hope this helps clarify! Let us know if you have any questions!

Thanks!

Get the most out of your LSAT Prep Plus subscription.

Analyze and track your performance with our Testing and Analytics Package.